Đến nội dung

Hình ảnh

$\sum \frac{a}{\sqrt{(1+a^{3})(1+b^{3})}}\geq \frac{4}{3}$


  • Please log in to reply
Chủ đề này có 2 trả lời

#1
Haton Val

Haton Val

    Hạ sĩ

  • Thành viên
  • 68 Bài viết

Cho a, b, c >0 thỏa abc=8 . CMR : $\sum \frac{a}{\sqrt{(1+a^{3})(1+b^{3})}}\geq \frac{2}{3}$


Bài viết đã được chỉnh sửa nội dung bởi Haton Val: 03-07-2017 - 11:50

$\sum_{x=7}^{18}x^{2}=2018$


#2
iloveyouproht

iloveyouproht

    Trung sĩ

  • Thành viên
  • 164 Bài viết

Cho a, b, c >0 thỏa abc=8 . CMR : $\sum \frac{a}{\sqrt{(1+a^{3})(1+b^{3})}}\geq \frac{2}{3}$

Bđt sai vs đ c=0,02 còn a=b=20 Bài này b chắc chế từ :

 

Cho a, b, c >0 thỏa abc=8 . CMR : $\sum \frac{a^{2}}{\sqrt{(1+a^{3})(1+b^{3})}}\geq \frac{4}{3}$  Đúng không ?


Bài viết đã được chỉnh sửa nội dung bởi iloveyouproht: 11-07-2017 - 16:30

Trước khi muốn bỏ cuộc, hãy nhớ lý do vì sao bạn bắt đầu…

________________________________________________

 

Kẻ thất bại luôn nhìn thấy khó khăn trong từng cơ hội...

Người thành công luôn nhìn thấy cơ hội trong từng khó khăn...  ~O)

-----------------------

My facebookhttps://www.facebook...100021740291096


#3
AnhTran2911

AnhTran2911

    Thượng sĩ

  • Thành viên
  • 230 Bài viết

Áp dụng BĐT $\frac{1}{\sqrt{x^3+1}}\geq{\frac{2}{x^2+2}}$

Nên ta có $\frac{a^2}{\sqrt{(1+a^3)(1+b^3)}}\geq{\frac{4a^2}{(a^2+2)(b^2+2)}}$

Do đó BĐT cuối cùng BĐTĐ cần CM: $2(a^2+b^2+c^2)+a^2b^2+b^2c^2+c^2a^2\geq{72}$ 

Đúng theo AM-GM vs giả thiết $abc=8$

ĐPCM


Bài viết đã được chỉnh sửa nội dung bởi AnhTran2911: 11-07-2017 - 17:00

        AQ02

                                 





0 người đang xem chủ đề

0 thành viên, 0 khách, 0 thành viên ẩn danh